From Wikipedia, the free encyclopedia

Questions from Heimstern

My questions are kind of nitty-gritty, but I'm not looking for really specific answers as much as trying to see your thought process and approaches to the issues.

1. What is your philosophy on how to handle edit warriors? Under what circumstances should the Committee ban users who continually edit war, and when should they use lesser sanctions, such as paroles or editing restrictions? What factors should the Committee consider in deciding what sanctions are appropriate?

I think that how to handle edit warriors is a bit wrapped up in question 2, so I'm going to answer them jointly. First, I think that the line between edit warring and heated but productive editing is not always easy to draw. Also, many people use different meanings of edit warring. For example, I've encountered at least one administrator who seemed to think that reverting to what was a clear consensus could constitute edit warring(although this problem could more be connected to what constitutes consensus than what constitutes edit warring). Obviously, edit warring can be a problem, and is more serious when accompanied by POV pushing and other problems. It is hard to make a generalization but I would say that uncivil editing, edit warring and POV pushing all combine in a non-linear fashion the severity of a problem (if that analogy doesn't make sense let me know and I'll expand on it). Two important reason that these need to be considered together are that 1) the combination of edit warring and associated problems can be much more stressful for other editors than edit warring or personal attacks by themselves and 2) editors are more likely to be incorrigible if they engage in multiple forms of problematic behavior. As to the second part of the question about what sanctions the Committee should consider I think in general each case should be decided individually. I will note that paroles and probations do not often seem to work effectively(there may be some Confirmation bias here since one doesn't pay much attention to the cases that do work) and so I would prefer more direct editing restrictions such as article or topic bans.

2. What about uncivil editors (including those making personal attacks)? What factors should the Committee consider in deciding whether and how to sanction them?

I've answered this question mainly in question 1, so I'll merely say that the nature, extremity and context of personal attacks is relevant. More severe or deliberately hurtful personal attacks are obviously more problematic, and there is a scale of personal attack whereby even a single attack can demonstrate that someone is not mature enough to be a member of this community.

3. When should an administrator be desysopped? In particular, how should a sysop's failings be weighed against his or her useful administrative actions, and when do the failings merit removal of adminship? When, if ever, is it appropriate to use a temporary suspension, such as was used in Wikipedia:Requests for arbitration/Jeffrey O. Gustafson?

My opinion about temporary desysopping has been in a state of flux for some time. First, we have a shortage of administrators. Many basic admin duties frequently have long backgrounds and we have one of the lowest rates of admins to active users of any Wikimedia project, so removing admins should be done with caution. I think that in general, barring extreme circumstances, admins should not be desysopped for a single action or set of actions within a short time span. We all make mistakes and simply because one admin made a single big mistake or made a mistake involving a matter that was highly emotion or drama-infested does not make the admin's activity more severe. However, repeated abuse of the admin tools(such as wheel-warring or use of the tools to gain advantage in content disputes), or repeated demonstration that an admin has poor judgment should be grounds for desysopping. Temporary suspensions may be useful as wake-up calls for otherwise good admins who have made a few bad judgments, but I don't see any compelling reason to expect that they will often work.

4. Under what circumstances should the Committee consider an appeal of a community ban?

Only if there is not a clear consensus for the ban. There is no point if the community has clearly decided that they cannot work with someone that the ArbCom should try to override their consensus. Very often this has been as there being no admin willing to unblock. I'm not sure that I am happy with that rule; as the number of admins grows the chance that someone can find one admin willing to unblock a bad user goes up. I'd therefore be willing to consider in cases where a single admin is willing to unblock but all other admins would likely block to be a possible community ban.

5. Two recent cases, Wikipedia:Requests for arbitration/Allegations of apartheid and Wikipedia:Requests for arbitration/THF-DavidShankBone, were dismissed with no decision made after the Committee had been unable to come to a decision concerning wrongdoing or sanctions. In both cases, the arbitrators seem to have felt that the cases' issues were no longer current, either because the community had resolved the issue or because a participant was no longer active at Wikipedia. Now, consider a similar situation in which the Committee cannot agree on finding concerning user conduct or on appropriate sanctions, but in which the case issues are clearly current. What should be done in such a case?

I have strong opinions about this case, since in the first one I was listed as an editor who had possibly engaged in problematic behavior. In remedy 6 (which was not passed) I was listed as an editor who "voted to delete the allegations of Israeli apartheid article, largely on principle, after having earlier voted to keep the allegations of Brazilian apartheid article. Given the circumstances, the only reasonable explanation for this voting pattern is that the editors in question were attempting to prove a point regarding the allegations of Israeli apartheid article." I wasn't following the case that closely and so wasn't even aware of this point until someone thankfully pointed this out on my user page. I contacted FloNight and pointed out to her that my opinions on the various AfDs had changed overtime as could be seen by a number of edits such as this one. That prompted FloNight to oppose the finding and following her lead others opposed as well. However, since no clear findings were made, I felt like my honor had been called into question. Furthermore, and much more seriously, in both the Apartheid case and the THF-Shankbone case the "solutions" that occurred were to a large extent imperfect. I suspect that if the ArbCom had been more timely more effective results may have occurred.
If the second situation occurs, the ArbCom is simply going to have to keep hammering away at the matter until they come up with something, or barring that find some clever third path. That is of course easy to say when I'm not actually on the committee. I hope that no such situations come up, but if they do arise, I really don't see much of a solution other than that proposed above.

Thanks for your consideration. Heimstern Läufer (talk) 00:26, 13 November 2007 (UTC) reply

Questions from east718

1. Do you feel that the Arbitration Committee takes too long to close cases? Or do you feel that they act too hastily and some important facets of cases occasionally fall through the cracks? Either way, what will you do to remedy it?

I think I've mainly answered this in my ArbCom statement and replies to Heimstern above. The only matter is the cracks falling through issue. The only way to handle that is for users to speak up when it looks like something is going to be missed. As the cliche goes, if you see something, say something.

2. Can you give some examples of proposed principles, findings of fact, or remedies on voting subpages that you disagree with? How about some proposals that actually passed? If you consider any completed arbitration cases to be failures in their intent, scope, or remedy, could you please name them and your reasoning why?

The first part of this question I answered in above in Heimstern's question involving Apartheid. If you want other examples, I'd also point to Finding of Fact 3 in Wikipedia:Requests for arbitration/Agapetos angel. The ArbCom found that "There has been significant tendentious editing to these articles by others who oppose the creationist point of view. Other edit warriors besides Agapetos angel include Duncharris, Guettarda, Jim62sch, and FeloniousMonk" I think, especially in retrospect, this was incorrect or at best inaccurate. I doubt that any of the editors's edits (aside from possibly a handful from Jim who was a new editor and so didn't know much about Wikipedia policy) would rise to being considered "tendentious" were it not the fact that the matter had already been brought to the ArbCom's attention. Furthermore, any problematic editing that did occur occurred in reaction to much worse editing by Sarfati et al. This reflects a general problem I've noticed with the ArbCom that they very often go for a pox on both houses and only rarely consider the possibility that only one side is editing so problematically as to deserve ArbCom's attention.

Thanks, east.718 at 01:13, 11/13/2007

Question from Cla68

Have you successfully nominated any articles that you've heavily edited for Featured or Good Article status? Cla68 03:09, 13 November 2007 (UTC) reply

No. I've helped get a few articles to featured status (such as Intelligent design) but have not nominated any of my own articles. I'm also a bit puzzled as to what this has to do with running for ArbCom. JoshuaZ 18:51, 13 November 2007 (UTC) reply

Question from Anonymous Dissident

By submitting a candidacy for the December 2007 Arbitration Committee Elections, you are indicating your commitment to Wikipedia, and your belief in its continuance and worth as a project. What do you personally see for the future, both near and distant, of Wikipedia as a collaborative effort to bring free knowledge to the planet, and what are your feelings in regards to the Arbitration Committees relation to the successful endurance of the quality and credibility, among other aspects, of Wikipedia, and of Wikipedia itself? -- Anonymous Dissident Talk 06:43, 13 November 2007 (UTC) reply

A minor nitpick with the question before I answer it. While candidacy for ArbCom does indicate that the person believes that the project is worthwhile, it isn't obvious to me that such candidacy necessarily indicates belief that the project will continue indefinitely. So to be clear, I think that the project should and will continue for the foreseeable future. We have many rival projects now, such as Wikinfo, Citizendium, Conservapedia and Dkosopedia. Despite this we are orders of magnitudes larger than any of these projects. While the argument can be made that we have an early-mover advantage I think a large part of our success stems from our commitment to NPOV and our open nature.
Now, to answer the main question: I think in the long run, Wikipedia is going to need to adopt some sort of stable version or flagged version policy, but I doubt that will change the general attitude in the public about Wikipedia. Those who dislike it will continue to dislike it, hose who rely on it uncritically will continue to get burned, and those who use Wikipedia carefully will continue to benefit. The Arbitration Committee can in my view do very little to help the general endurance of quality and credibility on Wikipedia. These issues are judged more based on the status of many articles as a whole. Even during the worst Wiki-drama the vast majority of editors continue plugging away writing articles, often oblivious to any claimed crisis. All of that said, Wikipedia's credibility and quality are based to a large extent on Wikipedia's openness and transparency. The ArbCom can help by promoting transparency and openness. Furthermore, on occasion Wikipedia conflicts do make their way into the media or other fora. In those cases, how the ArbCom handles those cases can have a direct impact on th public perception of the project. Finally, slow resolution of ArbCom cases or the issuing of sanctions that result in increased drama decrease the time spent by good editors and administrators from editing and protecting the encyclopedia; therefore, the ArbCom can help by resolving disputes quickly with an eye to the long-term and short-term consequences of their decisions. JoshuaZ 19:07, 13 November 2007 (UTC) reply

Questions from jd2718

ArbCom practice: 1. To what degree should ArbCom look at and treat administrators differently from non-administrators?

This is a difficult question, and I haven't made up my mind on it. Clearly there is one substantial difference: removal of tools is a possible option when dealing with a problematic admin. Thus, the ArbCom has that minimum additional issue when dealing with admins. There is an argument to which I am sympathetic that admins should be held to a higher standard than non-admins since admins should know better and should be role models for the community and must be people who can be trusted to act in the best interest of the community. This however need to be balanced by the consideration that admins are here for long amounts of time and are often under extended stress including serious campaigns of harassment and trolling. I'm not sure where these considerations balance out precisely.

2. Disputes over nationalist conflicts involving multiple editors make up a large chunk of ArbCom business. Why? Do these topics, articles, or editors need to be treated differently in some way by ArbCom?

The reason that nationalist disputes take up a lot of time is because there are many nationalists with strong feelings. This is understandable. We've had arbitrations about highways, and spoiler tags, and all sorts of things that don't at the end of the day matter much. But nationalism is different; people die every day fighting for or against nationalist causes. This is similar to why we've had many religion related RfArs. When people believe that what they are doing will determine the eternal fate of souls or prevent humanity to fall to a dark age of Christian fundamentalism/Islamic extremic/atheistic evolutionists etc. there is much more of an external desire to push ones POV. I'm not sure that these situations need to be treated differently, but I do think it may help to keep in mind that nationalists are much less likely to stop POV pushing or see that their POV is not NPOV than other groups. It may also be helpful to keep in mind that precisely because the stakes are so high Wikipedia has an extra obligation to people to make sure that our articles on nationalist conflicts are truly NPOV.

Views/experience: 1. If you were granted the power to change exactly one WP rule, policy, guideline, or practice, would you? Which?

I'm happy with most of our policies and guidelines but there is nothing major that I think needs to be changed with incredible urgency. I'll name three that I'm concerned about. This will no doubt annoy people who favor all three policies but such is life.
First, I think the fair-use guidelines could possibly use some tweaking to allow slightly more fair-use, but not much can be done with that without a change at the foundation level (and making any concrete proposals in this regard would require me to think about the issue a lot more than I have).
Second and more concretely, I'd like to see the BLP courtesy deletion changed so that it explicitly could not apply to willing public figures. People who have become involved in politics or other areas to the point where they are a public figure do not have a right to request courtesy deletion since they have willing opened themselves up to public scrutiny. Furthermore, not having articles on such people is obviously depriving people of coverage.
Third, I'd like to consider our inclusion policies (especially WP:N) to allow some degree of inherited notability, especially for characters in books or other works that are notable. Not so much as to make us automatically have an article on each one, but enough such as that we might not WP:FICT as strictly.

2. Can you point to a dispute (could have been at ArbCom or Mediation, or even on a talk page) that you've gone into (as an involved party or 3rd party) with a strong opinion, but had that opinion change in the course of discussion?

This is difficult since I don't hold strong opinions on many things on Wikipedia (indeed, if you look at many AfDs and DRVs you will see me repeatedly strike-out my previous comments as I change my position). One recent example where I had what might have been a strong opinion that changed was in the Martinphi-SA arbitration case. As you can see at the work shop page I strongly thought that SA's sockpuppetry while unfortunate was not abusive. I changed my mind when the evidence showed him using multiple accounts at once to change the tenor of a discussion.

Electing arbitrators: 1. Why did you enter this process over a week after it started? (I know it sounds accusatory, really isn't meant that way) Is the extended nomination period somehow unfair? Without specific reference to yourself or other candidates, what qualities, characteristics, or experiences do you think we should be looking for in an arbitrator?

I entered the nomination process a weak after it started because I wasn't sure I was going to run. I wasn't considering a run seriously until about a week before the nomination period opened when I then started receiving multiple emails saying that I should consider running. I then bounced the idea off of various users I trust and consulted various people in meatspace about whether they thought I had time for it. I then spent some time crafting my nomination statement which also took up 2 or 3 days.
As to the second part of the question- I don't see what could be unfair about the nomination period. If you could expand on the concern I might be able to answer it in more detail.
As to the third part, I think that important qualities in an arbitrator are patience, a thick skin, and experience with dispute resolution and various sorts of Wikipedia experience. One important form of experience is OTRS and related work. A number of candidates such as Deskana and Phil Sandifer have extensive experience with that which I see as a very good quality in an admin. I have experience informally helping when OTRS related issues have come on articles I already have on my watchlist or in a few other circumstances.

2. Last year the community nominated what looked like a solid bunch of Arbitrators. Yet 10 months later it turns out that several had very spotty levels of ArbCom activity. Do you think that this was at all predictable? And if so, how?

I'm not sure how to answer this. It might be possible to look at the less active arbitrators and pick out features they have in common, but there aren't any that stand out to me. Working as an arbitrator is clearly exhausting work that gets little thanks, and I suspect that in some cases the arbitrators were not as ready for it as they thought they would be. I don't know any way to foresee which arbitrators this will be a problem with.

You: 1. I've seen it written that to be a good arbitrator, a WPian needs to first be a good editor. Do you agree or disagree with the sentiment? How do you consider your own editing?

I'm not sure I agree or disagree with the sentiment. What constitutes a good editor? Most definitions of being a "good editor"(and I suspect that implied in the question) focus on work in mainspace. One definition of a good editor is an editor who has brought many articles up to good or featured status. Since ArbCom rarely makes content decisions [1] that shouldn't matter much. On the other hand, if someone has not made many good edits to a large variety of article they will likely not have the breath of experience necessary to understand some of the issues in front of the ArbCom. Another definition of a good editor is someone who has edited many controversial articles and helped people come to consensus about them. That might be more helpful for what an arbitrator does. There are other possible definitions of a "good editor" that focus less on mainspace but if we include them the number of possible definitions rapidly expands. Feel free to add a follow-up question with a more specific definition of good editor and I'll try to answer it.

2. Are you 15? 25? 55? Are you a student? Do you have an occupation that lends itself to allowing you time to be involved in ArbCom?

I am a student. I'm close to 25. I don't however think that an editor's age has any bearing on whether the editor would make a good arbitrator. Look at what the editor has done on Wikipedia. Look at the editor's statement and the editor's answers to questions. Judging by age makes in meatspace where we don't have the ability to examine all of someone's qualities in detail so we need to use somewhat reliable proxies. Wikipedia is transparent so we don't have much of that issue. As to the second part of the question, yes my occupation(student) does lend me to have time to be involved in ArbCom.

Thank you for indulging so many questions.

Question from Ultraexactzz

Best wishes in your candidacy, and in your tenure on the committee should you be elected. I'm asking this question to most of the candidates, so I apologize in advance if you've already answered a similar question from another editor.

Some background. I was an avid reader of the encyclopedia until December 2005, when I decided to begin editing. I had started to delve into the workings of the project, reading about AfD's and the ANI and, most interestingly, the work of the Arbitration Committee. When elections came around in December 2005/January 2006, I thought that a fresh perspective might be of value to the committee. So, in my haste to pitch in, I made my 13th edit (!) by nominating myself to the Arbitration Committee.

Needless to say, it did not go well.

However, I did find some editors who supported my candidacy on moral grounds, offering encouragement and concuring that a different perspective was of value in the committee's work. Looking back, it got me thinking, as this round of elections begins: What is the most valuable trait for an arbitrator? Your statement and answers to other questions will address this at length, I'm sure, but if you had to distill the essence of being an effective arbitrator into one word, what would that word be? ZZ Claims ~ Evidence 13:18, 13 November 2007 (UTC) reply

Josh doesn't think there is a single word to distill it into aside from illeism. Joking aside, I don't think there's a good single word, but if I had to make a small list I'd say 1) patience(which I have a lot of), 2)attention span (which I'll be honest I don't always have the best of), 3) experience with Wikipedia (which I have a lot of).

Question from Wikidudeman

In my experience, many larger arbitrations seem to suffer from the fact that the arbitrators do not spend as much time on examining the evidence and statements as they should be spending. Examples of problems that arise would be proposals not being used or relevant issues not even being addressed. This is probably due to the large backlog and caseload. What would you do to ensure that all arbitrations are ended efficiently and fairly and that all issues and concerns are addressed and all needed remedies met? Wikidudeman (talk) 14:48, 13 November 2007 (UTC) reply

First, let me disagree with one premise of the question: "proposals not being used" is not necessarily evidence that the ArbCom wasn't paying enough attention. For any given ArbCom at least the half the proposals in the workshop are simply bad. And for each good proposal there are often multiple variations of it proposed. Now, issues not being addressed is a more serious problem which does in many examples due to the arbitrators not examining the evidence sufficiently. As to this, I've already addressed in my statement and in prior questions some steps I'd take. To expand on that, I think one good idea is for individual cases for a few Arbitrators (maybe 2 ) to agree to focus in particular on that case and then to give the other arbitrators some summaries and pointers of what to look at. This would work more efficiently than having each arbitrator look at each detail of every single case. Of course, one will need to be careful to make sure that the few looking in detail don't miss anything, but it should help matters. (Disclaimer, I actually don't know much about how the ArbCom is currently dividing work informally but the impression I get is that they aren't doing much in that regard).

Question from Ragesoss

In the Wikipedia context, what is the difference (if any) between NPOV and SPOV (scientific point of view)?-- ragesoss 17:19, 13 November 2007 (UTC) reply

This is a complicated question that I've given a lot of thought to. Unfortunately, the thoughts aren't very organized, so I'll see what I can do. A quick preamble: WP:SPOV is a long rejected proposal to write from SPOV rather than NPOV. However, when people discuss the differences between them they aren't generally paying much attention to what that proposal said but rather to the more general notion of SPOV. Now moving on:
Most us agree that there is a difference between the two. I don't think that the difference is in general clear. For some topics, like say Reaction (physics) or electron, SPOV and NPOV versions of the article will be identical. However, for some topics, SPOV is meaningless. For example, it doesn't make sense to speak of an SPOV version of American Idol. And then there are articles where the SPOV version will differ from a NPOV, for example Homeopathy. An SPOV version of that article would like say something along the lines of "Homeopathy is a long debunked idea. It might be interesting for sociologists and psychologists to investigate why the topic keeps being brought up" followed by a few sources. Obviously, an article that said that would not be NPOV. On the other hand, there are issues of undue weight, and so for example Evolution which has an overwhelming scientific consensus gives only a small mention to views which disagree with it.
So where is the line for when SPOV does in fact equal NPOV? Suppose we had a reliable source that mentioned a person who thought that all the Wolbachia in the world was linked using tachyons to communicate as part of a hivemind working with the Illuminati and sentient Jewish gyroscopes to use alien device made by Xenu found in the Bermuda triangle to go back in time and frame Osama bin Laden for 9/11. [2] Would we need to mention it in the article Wolbachia? I think not. Well then, what if the one reliable reference was the New York Times or the Wall Street Journal? What if the New York Times did an article about this person? What if the person who had the idea was otherwise notable? Clearly for every major biology topic we don't need mention when we mention evolution "oh and by the way there are people who don't believe this" but we'd likely be remiss if we didn't have some mention at Evolution that there are people who don't accept it. These aren't easy issues and I'm not sure where to draw the line. There may not be a clear line.
However, the interaction is even more complicated because it isn't always clear if there is a single SPOV or an SPOV at all. As Durova noted in her well-thought out answer to the same question, it isn't always clear that there is an SPOV. As Durova noted, doctors in different countries can have radically different but still scientific attitudes about various symptoms and diseases. [3]
What Durova's brings up is not the only problem of this sort. Consider Electronic voice phenomena(EVP), where no serious scientists have bothered investigating the matter because from the general meta-patterns(I'd say meta-paradigm but that's really pretentious) of how modern science sees how the world works, the idea is improbable and a few moments of critical thinking only make it seem even less likely. So there are not many reliable sources to present the "SPOV" and yet we have a clear idea of what the SPOV is: Something close to "This is a waste of time. If these entities can change static to make hard to understand signals why can't they simply manipulate individual bits? There is no satisfactory explanation for why this supposedly works for both digital and analog static. For further information please see Pareidolia." So when people mean SPOV they don't really mean SPOV in the sense of what science has actually said about the matter, but rather what they expect science would say if scientists investigated the topic after they apply methodological naturalism and certain types of critical thinking. At some level this is almost unfortunate; the article on EVP needs to be almost completely credulous because scientists haven't taken the subject seriously at all and so has only two short paragraphs devoted to the SPOV. So precisely because the SPOV is so negative ( to the point of dismissal) the article has almost no sources to present about the SPOV. If one looks at EVP and Intelligent design one notes an interesting pattern which holds for many fringe science and pseudoscience topics; a topic doesn't get a response from the scientific community unless it has made enough inroads somewhere (either with scientists or lay people or claimed experts) that scientists consider it either responding to, investigating or debunking. This means that our policies at some level force us to present less popular fringe subjects more credulously than more popular fringe subjects.
There's another issue; even for those people today who don't favor science much, there's a feeling that science is a dominant viewpoint that needs to have more attention paid to it than others. Thus, when Biblical literalists contrast their viewpoint they contrast it to the scientific consensus, not to the extremist Hindus espousing a literal interpretation of the Vedas or to that of the followers of Immanuel Velikovsky. So one might almost say that NPOV favors a large amount of weight given to the scientific viewpoint because all alternative viewpoints(for lack of a better term) consider some sort of SPOV to be their major competitor. I'm not sure I agree with that argument, but it is worth considering.
However, scientific viewpoints do and should get a lot of weight for one simple reason: science is very good at producing reliable sources. Since the primary method of deciding how much weight to give something is how many reliable sources we have on a topic, this naturally gives science a large amount of weight. But even this raises questions: for example, Answers in Genesis and the Institute for Creation Research have both constructed what they claim are scientific journals, such as the "Journal of Creation" which has pro- Young Earth creationism articles which they claim are peer reviewed. Critics would likely label this work Cargo cult science. Now, are we as Wikipedia to treat these articles the same way we would treat mainstream scientific work in journals? Right now the consensus is not to, but for every fringe or pseudoscience group that makes a specialized journal the question of how to treat their journals come up. And there cases less clear cut than those for the Young Earth Creationists such as those that support Cold fusion.
So where does this all put us? There are four points that we should take away: 1) SPOV is not NPOV 2) What is SPOV is complicated. 3) How SPOV interacts with NPOV is complicated. 4) In general, when there is a coherent SPOV it should and will generally be given much weight but precisely how much weight is unclear.

Points of View: When does including "notable" points of view become problematic for NPOV?

When I first came on to Wikipedia a year and a half ago the project was more centered around "Just the facts" - articles were more crafted around the who, the what, the when and the where, with some emphasis on the why. Of late, the why has taken on a dominant role in articles on contentious issues, with each side in the political spectrum putting forth their own "notable" mouthpiece to spin what the who, the what, the when and the where means. Do you think this is a positive development? Do you think this is educational, or do you think it makes Wikipedia another platform for the dichotomized public debate--that there are two sides to every issues, and two views--that is prevalent in American society?--David Shankbone 18:27, 17 November 2007 (UTC) reply

I'm going to disagree somewhat with the premise. For many contentious articles even the facts are in dispute. See for example some of the disputes that occurred on Beit Hanoun November 2006 incident. To some extent this is a sign of Wikipedia's maturity and is a positive development. Many topics now have the articles flushed out better and we've reach consensuses about how to present matters where there is factual disagreement. Some policies like the strengthened BLP make it more clear what to do when there are questionable facts. To some extent this phenomenon is a sign that Wikipedia is more in the public eye, and so partisan groups understand that if they can control Wikipedia's presentation of the why they can influence the hearts and minds of many. I think Wikipedia needs to be careful not to aggravate what you call the dichotomized public debate. However, Wikipedia must reflect what reliable sources say. If all the reliable sources say simply "Some experts say X. Some experts say ~X and say that anyone who says X is stupid or lying" there isn't much we can do about it. Wikipedia can only reflect what the reliable sources say. At the same time, NPOV should help people understand all sides, when there are multiple sides or understand continua when there are continua of opinions.

Question from I

  1. What, if anything, do you believe is wrong with the current arbitration process, and/or the committe? This includes anything related to the committee and its actions. If appointed, what do you intend to do to resolve these issues? I (talk) 17:51, 18 November 2007 (UTC) reply
I think I've answered this well-enough in my statement as well as my replies to Heimstern and Wikidudeman above. JoshuaZ ( talk) 03:03, 24 November 2007 (UTC) reply

Questions from Piotrus

1. Do you think an arbitrator should be active in all cases he has no conflict or interests in?

No. Activity in all cases one can be involved in is a likely recipe for burnout.
If the arbitrator is active, should he be expected to comment in workshop / arbcom discussion pages?

Define "expected" it could likely help but there's no point in commenting if one doesn't have anything useful to add that hasn't already been said. Transparence is good but me too comments are not that useful.

  1. Do you think some editors should be more equal than others? I.e. should incivility of experienced editor - one who registered years ago and wrote or contributed to many articles - be treated differently from incivility of a relative newcomer?
I've answered this question to a limited extent with my answer to jd2718's question 1. Although he focused on administrators the basic thrust of the answer applies to all experienced editors with one additional point: if an editor has been around for a long time and has been consistently violating WP:CIVIL or any other policy or guideline the likelyhood of rehabilitation is low.
  1. How can WP:CIV and similar issues be enforced? Should they be enforced as efficient as 3RR?
CIV can be enforced by blocks, paroles and removals of commenst as we do so already. Unlike 3RR, WP:CIV is not always clear cut nor cut. Therefore it can be enforced as efficiently or mechanically as 3RR (indeed, mechanical enforcement of 3RR isn't often a good thing either).

-- Piotr Konieczny aka Prokonsul Piotrus | talk 01:47, 23 November 2007 (UTC) reply

Question from AniMate

Arbitration is the last step in dispute resolution. However, first and foremost, we are here to work on an encyclopedia. Editing and adding to the project should be everyone's first priority. Can you point out some of your recent mainspace contributions that you are most proud of? AniMate 12:06, 23 November 2007 (UTC) reply

My answer to this question will likely be not that impressive since I tend to poke around mainspace, not concentrating on any single article for that long. One article I've been recently involved with was Francis J. Beckwith‎ where the article subject has recently shown up and objected to some of the language and I've been working to resolve those. I've also been working on xkcd replacing original research about the comic with cited references. As you can see by checking my contribution list, the editing I've done is varied. JoshuaZ ( talk) 01:22, 26 November 2007 (UTC) reply

Question from Wanderer57

Based on ‘Request for comment on user conduct’ processes that you have followed closely, how would you rate them in terms of fairness to the accused?

(Just to be clear. Some candidates wondered if my question was "aimed at them". I'm asking all candidates the same generic question; it is not aimed at anyone.)

Thanks, Wanderer57 ( talk) 15:14, 24 November 2007 (UTC) reply

First, the RfC process is an attempt to get people to listen to criticism and to reform, so referring to the subject as the "accused" is not helpful. That out of the way: I'm divided about the RfC process. Sometimes is degenerates into what looks like mobs, and RfCs will sometimes draw out everyone who has a grievance with a given editor. However, when the community does support the subject's actions such as Wikipedia:Requests_for_comment/Bishonen it is made clear. A more serious issue is that the process rarely does anything. It is useful for taking some load off of arb-com, but it isn't much more than that. And I'm not convinced it really reduced the Arb-Com's load all that much. JoshuaZ ( talk) 01:35, 26 November 2007 (UTC) reply

Question from Cla68

So that it won't look like I'm targeting anyone in particular, I'm asking this question of all the candidates. Were you a recipient on the email list used by Durova to distribute her evidence used to wrongfully block !! as detailed in this ArbCom case? Cla68 ( talk) 00:50, 27 November 2007 (UTC) reply

I'm highly tempted not to answer this question as it is completely and utterly irrelevant and fosters unnecessary drama. Off-wiki communications are a standard part of Wikipedia discussions and are especially necessary when dealing with determined troublemakers like those who frequent WR and ED. That said, no I was not. I have not seen any serious wrong done here. Durova made an incorrect block, after some discussion the block was overturned. Almost every single admin has made at least one bad block (I've made at least 4). This matter is being blown out of proportion and is a complete waste of time.

Questions from Rschen7754

1. What are your views regarding debates such as WP:RFAR/HWY and WP:SRNC? (In terms of dispute resolution).

The ongoing highways disupute was one of the most amazingly lame disputes I've ever seen and wasted a tremendous amount of time. I didn't pay much attention to the dispute preciesely for that reason. However, my general impression is that the final result (the poll and related issues) handled at best could be handled a highly contentious dispute.

2. a) What is the purpose of a WikiProject? Do you believe that WikiProjects b) own articles or c) can enforce standards (such as article layout) on articles?

Wikiprojects exist to coordinate editing. I'm generally worried when wikiprojects attempt to own articles or ayyrmpy to enforce their own article layout standard. Note that this is distinct from a general consensus of editors involved with a type of article reaching a consensus about how layout and other issues should be standardized. That is in general fine, and if most of the editors involved in such a consensus are part of a wikiproject that's ok. However, Wikiprojects do not in general have any special power.

3. Do you believe that parent WikiProjects have the right to impose standards (such as article layout) on child WikiProjects? (Case in point: WP:USRD and its state highway projects)

See question 2 above. Since Wikiprojects have no special power to impose standards on article space it doesn't make sense to discuss them imposing article layout on their child projects.

4. a) What is your definition of canvassing? b) Does it include project newsletters or IRC?

Canvassing is a complicated issue and I'm not sure that I can give a short answer. I would suggest a definition along the lines of "Deliberately asking for people of one opinion to chime in in a discussion or vote without alerting people with opposing opinions."

However, this definition can be too broad on occasions. If for example a single editor said to another editor "let me know when you are running for admin" it wouldn't be canvassing to alert the editor in question. And if that happened with 2 or 3 editors it wouldn't be either. Where exactly one draws lines for this sort of thing isn't always clear.

5. a) In terms of vandalism and good faith but horrible edits, where do you draw the line? (scenario: an editor makes a mess of articles that cannot easily be fixed). b) Should blocks, protects, and / or rollbacks be in order?

As to a, it really depends on the circumstances. If it doesn't appear to be vandalism then communicating with the editor is probably a good idea. Blocks should not be used unless multiple attempts to communicate fail. Rollback might also be used if cleaning up would be very difficult otherwise. However, these are my personal opinions (and are vague at that); I don't claim that there's any real policy basis behind this. If someone else drew the line elsewhere I wouldn't have a problem with that.

Thank you. -- Rschen7754 ( T C) 06:58, 27 November 2007 (UTC) reply

Question from Blue Tie

1. Can/should Arbcom create wikipedia policy? Or develop a proposed policy for community vote?

Arbcom should not in general create policy. However, in practice the ArbCom does sometimes do so, for example deciding that certain types of conduct are unacceptable even when no policy said so. I'd prefer that this sort of policy making be kept to a minimum and would strongly not favor retroactive punishment for policy violation.

2. Do you intend to help create or propose wikipedia policy as an Arbcom member? -- Blue Tie 13:19, 30 November 2007 (UTC) reply

I have constructed policy issues in the past as a general Wikipedian and intend to do so in that capacity whether or not I am elected to the ArbCom. As I said before, ArbCom should not take part in writing policy unless absolutely necessary.

Question from SilkTork

How would you vote on this proposed principle: "While anyone may edit Wikipedia without the need to register, that meta-editing activities such as voting in an ArbCom Election are best protected by registering than by sleuthing". SilkTork * SilkyTalk 17:44, 1 December 2007 (UTC) reply

I disagree with this proposal in part and agree with it in part. First, there is no easy mechanism to create a registration process that would not be circumventable. Having the Foundation do so would be costly, unwieldy and get the foundation involved to an extent that could make it up unacceptably vulnerable from a legal perspective. Almost any registration system (such as having voters' real names on file somewhere) would also require people who registered to give up some of their privacy. Furthermore, our most common forms of voting is RfA and that had way too many people involved in it. Similarly, AfDs have way too many people for this to be practical. I also object to the labeling of "sleuthing" as "security by obscurity". We generally use so called "sleuthing" to pick up socks of known repeated disrupters. Many people have little tell-tails about how they write and edit. Letting such details in public is problematic since the people can then adjust. But in general, such methods when used properly and when backed up by checkuser and other evidence can be very useful. Now, given the much smaller number of people who vote in the ArbCom election, doing something similar restricted solely to people who want to vote might be more workable. But really insidious socks aren't likely to change something like the ArbCom results, but are more likely to do serious damage at AfD or other areas. JoshuaZ ( talk) 21:13, 13 December 2007 (UTC) reply

Questions from Irpen

The questions below refer to the issues of ArbCom's integrity and transparency that needs to be maintained despite the universally accepted view that certain things should remain private.

Mailing list

Arbitrator's private mailing list, known as Arbcom-l and the arbitrators only IRC channel may obviously include information that cannot be made public under any circumstances. Additionally, being aware of the intra-ArbCom communication may give case parties an obvious advantage over their opponents. Who do you think should have access to such a list besides current arbitrators whose community trust has been confirmed in election that took place within the last 3 years? Should it include users that where never voted on? Should it include users who were voted 4, 5 or more years ago? Should users who are parties of the case, comment on the case, present evidence on the case, be allowed to have read access to the list where the case is discussed by the decision makers?

Secret evidence and secret communication of arbitrators with non-arbitrators

What is your opinion about the parties of the case (or anyone) contacting arbitrators privately about the case? This is not an hypothetical issue and it has been brought up in past cases. The obvious drawback is that if charges are brought secretly, the accused cannot see them and respond. Would you support an amendment of the arbitration policy that would prohibit parties from writing to arbitrators privately in relation to the cases? Giving evidence that has to be private due to its sensitive nature would of course be exempted but should this be the only exception?

Recusals

Arbitrators who are parties of the case or have an involvement with the case parties that can reasonably be considered to affect their impartiality are expected to recuse. What involvement constitutes the ground for a recusal has traditionally been left to the arbitrators' own discretion, except for obvious cases when arbitrators themselves are case parties. While recused arbitrators, especially the case parties, are allowed to take an active part in cases, collect, present and discuss evidence at the case pages, the same way as ordinary parties, they retain the opportunity to read the thoughts of other arbitrators at Arbcom-L and respond to those privately. It is technically difficult to exclude arbitrators from communication on a case they are involved. But would you support a prohibition for such arbitrators to discuss the case with other arbitrators through the private communication channels, except when submitting evidence whose nature warrants non-publicity?

Community oversight over the arbitration policy

Policies are written by the community and not by the ArbCom. However, at some point the ArbCom made it clear that the arbitration policy is exceptional in this respect and that the ArbCom intends to control the main policy that governs its own action rather than be governed by the policy written by the community. Would you support returning the control of the ArbCom policy back to the community or should the ArbCom write its policy itself?

Question from Mrs.EasterBunny

As a member of ArbCom, would you place more emphasis on content or behavior? For example, in the http://en.wikipedia.org/wiki/Wikipedia:Requests_for_arbitration/SevenOfDiamonds case, there is voluminous discussion on whether SevenofDiamonds is really NuclearUmpf, but no discussion on what got NuclearUmpf banned in the first place. If SevenofDiamonds=NuclearUmpf, then this is a behavioral problem but doesn't have to be a content problem. If SevenofDiamonds edit was reasonable (I have not researched it) would it make a difference?

The above may not be the best example but it's one that I recently saw because I can't remember the parties involved in similar cases. On occasion, I have seen an editing admin block someone because of a dispute in editing an article that both of them are editing and the block seemed questionable because there is no overt POV. The blocked editor then probably feels the block is unjustified and creates a sock. Many times, people running for WP office will cite a clear cut case of someone with bad editing and bad behavior. However, what if there is good editing and improper block (which would point to admin misconduct about content), followed by sock creation justified because the block was improper (which would point to editor misconduct about behavior)? Does the first crime excuse the second? Or is the second one crime much more serious and punishable? (This is not an easy question because excusing the first crime by the admin would tend to increase the workload of ArbCom because it allows admin to do a lot with less oversight. However, excusing the second crime might seem to encourage socks). Mrs.EasterBunny ( talk) 20:18, 5 December 2007 (UTC) reply

Question from wbfergus

What is your position on the following?

  • A policy page has had a very active discussion for many months. All sides (loosely termed 'pro-change', 'anti-change' and 'issue-specific') of proposed changes have made their cases back and forth numerous times. The 'pro-change' group is mainly users, with a few Admins. The 'anti-change' group is mainly Admins (including those who helped write the policy over the years) and a few users. The 'issue-specific' group is a mixed collection of users and Admins, but mainly users. All three groups constitute around 40-50 people total, per announcements on the Village Pump and related policies, to garner more widespread community involvement either way.
  1. After numerous discussions, and comments over a span of several days to several weeks on specific issues, what should constitute a consensus? 60%, 75%, 90%, or unanimous approval?
  2. If around 75% agree to a change, is it appropriate for Admins (especially those who helped write the policy) to revert changes and protect the page from further edits against their approval?
  3. Is it appropriate for 6 or 7 Admins to more or less block changes to a policy through protection and reverts, when very active discussions have been ongoing and the majority of those participating constructively (not just saying "No" or "Oppose" without constructive comments) agree to changes?
  4. Would it be appropriate for such a policy page which does clearly have a disputed section to have a tag in that section stating that section is under dispute and to participate on the talk page?
  5. Should policies solely dictate acceptable and unacceptable content, behaviour, etc., or should they also define Wikipedia-specific terms and definitions (without stating so) that conflict with usage in different disciplines, or should such terms and definitions be more appropriately suited in a guideline linked to and from the policy?
  6. Do you agree that policies are meant for enforcement or 'enforceable actions', while guidelines are meant to give guidance?

wbfergus Talk 15:35, 12 December 2007 (UTC) reply

For the record, I feel that I need to close my questions to all candidates, as one of the editors in the above 'subject' has filed an ArbCom request. As such, it could be interpreted as unseemly or whatever for these issues to be addressed in this forum. I was in the process of cancelling my questions and replying in an RfC and the related ArbCom request when I had to leave to take my wife to a Dr. appointment, so pardon the delay in cancelling this. wbfergus Talk 21:15, 13 December 2007 (UTC) reply

References

  1. ^ I'm not going to pretend that the ArbCom never makes content decisions even though that is the standard line. The BLP/Badlydrawnjeff decision for example led to what amounted to essentially content decisions for example. Furthermore, when the ArbCom bans or restricts editors who have a certain POV on an article it is going to have content results albeit more indirectly.
  2. ^ I constructed this example from a variety of different conspiracy theories, each one of which I've seen mentioned in at least one reliable source with the exception of the gyroscope element which I borrowed from xkcd. My intent here isn't just humor. We need to be able to decide how much weight to give to subjects and people do inevitably include the giggle factor. If they can't take a subject seriously they will be less likely to give it as much weight in an article. This can lead to problems. I've seen AfDs where the sole cause of the AfD was the nominator considering the subject to be too ridiculous to merit an article. For the relevant xkcd see "Gyroscopes(#332)". xkcd. Retrieved 2007-11-23..
  3. ^ See Wikipedia:Arbitration_Committee_Elections_December_2007/Candidate_statements/Durova/Questions_for_the_candidate#23_17 for Durova's excellent and concise description of this problem.

Question from Pinkville

Wikipedia is a community that produces and maintains a (still-nascent) encyclopaedia. This community has particular social and political structures that define it and that, presumably, affect the character, quality, and depth of its encyclopaedic output. Can you briefly summarise some political and social aspects of the Wikipedia community that you consider important or noteworthy, that perhaps need to be challenged or developed? How does the structure of Wikipedia encourage or inhibit access to decision-making and issues of power/control? Or does any of that matter? And what are the implications for the Arbitration Committee and its members? Pinkville ( talk) 22:10, 12 December 2007 (UTC) reply

From Wikipedia, the free encyclopedia

Questions from Heimstern

My questions are kind of nitty-gritty, but I'm not looking for really specific answers as much as trying to see your thought process and approaches to the issues.

1. What is your philosophy on how to handle edit warriors? Under what circumstances should the Committee ban users who continually edit war, and when should they use lesser sanctions, such as paroles or editing restrictions? What factors should the Committee consider in deciding what sanctions are appropriate?

I think that how to handle edit warriors is a bit wrapped up in question 2, so I'm going to answer them jointly. First, I think that the line between edit warring and heated but productive editing is not always easy to draw. Also, many people use different meanings of edit warring. For example, I've encountered at least one administrator who seemed to think that reverting to what was a clear consensus could constitute edit warring(although this problem could more be connected to what constitutes consensus than what constitutes edit warring). Obviously, edit warring can be a problem, and is more serious when accompanied by POV pushing and other problems. It is hard to make a generalization but I would say that uncivil editing, edit warring and POV pushing all combine in a non-linear fashion the severity of a problem (if that analogy doesn't make sense let me know and I'll expand on it). Two important reason that these need to be considered together are that 1) the combination of edit warring and associated problems can be much more stressful for other editors than edit warring or personal attacks by themselves and 2) editors are more likely to be incorrigible if they engage in multiple forms of problematic behavior. As to the second part of the question about what sanctions the Committee should consider I think in general each case should be decided individually. I will note that paroles and probations do not often seem to work effectively(there may be some Confirmation bias here since one doesn't pay much attention to the cases that do work) and so I would prefer more direct editing restrictions such as article or topic bans.

2. What about uncivil editors (including those making personal attacks)? What factors should the Committee consider in deciding whether and how to sanction them?

I've answered this question mainly in question 1, so I'll merely say that the nature, extremity and context of personal attacks is relevant. More severe or deliberately hurtful personal attacks are obviously more problematic, and there is a scale of personal attack whereby even a single attack can demonstrate that someone is not mature enough to be a member of this community.

3. When should an administrator be desysopped? In particular, how should a sysop's failings be weighed against his or her useful administrative actions, and when do the failings merit removal of adminship? When, if ever, is it appropriate to use a temporary suspension, such as was used in Wikipedia:Requests for arbitration/Jeffrey O. Gustafson?

My opinion about temporary desysopping has been in a state of flux for some time. First, we have a shortage of administrators. Many basic admin duties frequently have long backgrounds and we have one of the lowest rates of admins to active users of any Wikimedia project, so removing admins should be done with caution. I think that in general, barring extreme circumstances, admins should not be desysopped for a single action or set of actions within a short time span. We all make mistakes and simply because one admin made a single big mistake or made a mistake involving a matter that was highly emotion or drama-infested does not make the admin's activity more severe. However, repeated abuse of the admin tools(such as wheel-warring or use of the tools to gain advantage in content disputes), or repeated demonstration that an admin has poor judgment should be grounds for desysopping. Temporary suspensions may be useful as wake-up calls for otherwise good admins who have made a few bad judgments, but I don't see any compelling reason to expect that they will often work.

4. Under what circumstances should the Committee consider an appeal of a community ban?

Only if there is not a clear consensus for the ban. There is no point if the community has clearly decided that they cannot work with someone that the ArbCom should try to override their consensus. Very often this has been as there being no admin willing to unblock. I'm not sure that I am happy with that rule; as the number of admins grows the chance that someone can find one admin willing to unblock a bad user goes up. I'd therefore be willing to consider in cases where a single admin is willing to unblock but all other admins would likely block to be a possible community ban.

5. Two recent cases, Wikipedia:Requests for arbitration/Allegations of apartheid and Wikipedia:Requests for arbitration/THF-DavidShankBone, were dismissed with no decision made after the Committee had been unable to come to a decision concerning wrongdoing or sanctions. In both cases, the arbitrators seem to have felt that the cases' issues were no longer current, either because the community had resolved the issue or because a participant was no longer active at Wikipedia. Now, consider a similar situation in which the Committee cannot agree on finding concerning user conduct or on appropriate sanctions, but in which the case issues are clearly current. What should be done in such a case?

I have strong opinions about this case, since in the first one I was listed as an editor who had possibly engaged in problematic behavior. In remedy 6 (which was not passed) I was listed as an editor who "voted to delete the allegations of Israeli apartheid article, largely on principle, after having earlier voted to keep the allegations of Brazilian apartheid article. Given the circumstances, the only reasonable explanation for this voting pattern is that the editors in question were attempting to prove a point regarding the allegations of Israeli apartheid article." I wasn't following the case that closely and so wasn't even aware of this point until someone thankfully pointed this out on my user page. I contacted FloNight and pointed out to her that my opinions on the various AfDs had changed overtime as could be seen by a number of edits such as this one. That prompted FloNight to oppose the finding and following her lead others opposed as well. However, since no clear findings were made, I felt like my honor had been called into question. Furthermore, and much more seriously, in both the Apartheid case and the THF-Shankbone case the "solutions" that occurred were to a large extent imperfect. I suspect that if the ArbCom had been more timely more effective results may have occurred.
If the second situation occurs, the ArbCom is simply going to have to keep hammering away at the matter until they come up with something, or barring that find some clever third path. That is of course easy to say when I'm not actually on the committee. I hope that no such situations come up, but if they do arise, I really don't see much of a solution other than that proposed above.

Thanks for your consideration. Heimstern Läufer (talk) 00:26, 13 November 2007 (UTC) reply

Questions from east718

1. Do you feel that the Arbitration Committee takes too long to close cases? Or do you feel that they act too hastily and some important facets of cases occasionally fall through the cracks? Either way, what will you do to remedy it?

I think I've mainly answered this in my ArbCom statement and replies to Heimstern above. The only matter is the cracks falling through issue. The only way to handle that is for users to speak up when it looks like something is going to be missed. As the cliche goes, if you see something, say something.

2. Can you give some examples of proposed principles, findings of fact, or remedies on voting subpages that you disagree with? How about some proposals that actually passed? If you consider any completed arbitration cases to be failures in their intent, scope, or remedy, could you please name them and your reasoning why?

The first part of this question I answered in above in Heimstern's question involving Apartheid. If you want other examples, I'd also point to Finding of Fact 3 in Wikipedia:Requests for arbitration/Agapetos angel. The ArbCom found that "There has been significant tendentious editing to these articles by others who oppose the creationist point of view. Other edit warriors besides Agapetos angel include Duncharris, Guettarda, Jim62sch, and FeloniousMonk" I think, especially in retrospect, this was incorrect or at best inaccurate. I doubt that any of the editors's edits (aside from possibly a handful from Jim who was a new editor and so didn't know much about Wikipedia policy) would rise to being considered "tendentious" were it not the fact that the matter had already been brought to the ArbCom's attention. Furthermore, any problematic editing that did occur occurred in reaction to much worse editing by Sarfati et al. This reflects a general problem I've noticed with the ArbCom that they very often go for a pox on both houses and only rarely consider the possibility that only one side is editing so problematically as to deserve ArbCom's attention.

Thanks, east.718 at 01:13, 11/13/2007

Question from Cla68

Have you successfully nominated any articles that you've heavily edited for Featured or Good Article status? Cla68 03:09, 13 November 2007 (UTC) reply

No. I've helped get a few articles to featured status (such as Intelligent design) but have not nominated any of my own articles. I'm also a bit puzzled as to what this has to do with running for ArbCom. JoshuaZ 18:51, 13 November 2007 (UTC) reply

Question from Anonymous Dissident

By submitting a candidacy for the December 2007 Arbitration Committee Elections, you are indicating your commitment to Wikipedia, and your belief in its continuance and worth as a project. What do you personally see for the future, both near and distant, of Wikipedia as a collaborative effort to bring free knowledge to the planet, and what are your feelings in regards to the Arbitration Committees relation to the successful endurance of the quality and credibility, among other aspects, of Wikipedia, and of Wikipedia itself? -- Anonymous Dissident Talk 06:43, 13 November 2007 (UTC) reply

A minor nitpick with the question before I answer it. While candidacy for ArbCom does indicate that the person believes that the project is worthwhile, it isn't obvious to me that such candidacy necessarily indicates belief that the project will continue indefinitely. So to be clear, I think that the project should and will continue for the foreseeable future. We have many rival projects now, such as Wikinfo, Citizendium, Conservapedia and Dkosopedia. Despite this we are orders of magnitudes larger than any of these projects. While the argument can be made that we have an early-mover advantage I think a large part of our success stems from our commitment to NPOV and our open nature.
Now, to answer the main question: I think in the long run, Wikipedia is going to need to adopt some sort of stable version or flagged version policy, but I doubt that will change the general attitude in the public about Wikipedia. Those who dislike it will continue to dislike it, hose who rely on it uncritically will continue to get burned, and those who use Wikipedia carefully will continue to benefit. The Arbitration Committee can in my view do very little to help the general endurance of quality and credibility on Wikipedia. These issues are judged more based on the status of many articles as a whole. Even during the worst Wiki-drama the vast majority of editors continue plugging away writing articles, often oblivious to any claimed crisis. All of that said, Wikipedia's credibility and quality are based to a large extent on Wikipedia's openness and transparency. The ArbCom can help by promoting transparency and openness. Furthermore, on occasion Wikipedia conflicts do make their way into the media or other fora. In those cases, how the ArbCom handles those cases can have a direct impact on th public perception of the project. Finally, slow resolution of ArbCom cases or the issuing of sanctions that result in increased drama decrease the time spent by good editors and administrators from editing and protecting the encyclopedia; therefore, the ArbCom can help by resolving disputes quickly with an eye to the long-term and short-term consequences of their decisions. JoshuaZ 19:07, 13 November 2007 (UTC) reply

Questions from jd2718

ArbCom practice: 1. To what degree should ArbCom look at and treat administrators differently from non-administrators?

This is a difficult question, and I haven't made up my mind on it. Clearly there is one substantial difference: removal of tools is a possible option when dealing with a problematic admin. Thus, the ArbCom has that minimum additional issue when dealing with admins. There is an argument to which I am sympathetic that admins should be held to a higher standard than non-admins since admins should know better and should be role models for the community and must be people who can be trusted to act in the best interest of the community. This however need to be balanced by the consideration that admins are here for long amounts of time and are often under extended stress including serious campaigns of harassment and trolling. I'm not sure where these considerations balance out precisely.

2. Disputes over nationalist conflicts involving multiple editors make up a large chunk of ArbCom business. Why? Do these topics, articles, or editors need to be treated differently in some way by ArbCom?

The reason that nationalist disputes take up a lot of time is because there are many nationalists with strong feelings. This is understandable. We've had arbitrations about highways, and spoiler tags, and all sorts of things that don't at the end of the day matter much. But nationalism is different; people die every day fighting for or against nationalist causes. This is similar to why we've had many religion related RfArs. When people believe that what they are doing will determine the eternal fate of souls or prevent humanity to fall to a dark age of Christian fundamentalism/Islamic extremic/atheistic evolutionists etc. there is much more of an external desire to push ones POV. I'm not sure that these situations need to be treated differently, but I do think it may help to keep in mind that nationalists are much less likely to stop POV pushing or see that their POV is not NPOV than other groups. It may also be helpful to keep in mind that precisely because the stakes are so high Wikipedia has an extra obligation to people to make sure that our articles on nationalist conflicts are truly NPOV.

Views/experience: 1. If you were granted the power to change exactly one WP rule, policy, guideline, or practice, would you? Which?

I'm happy with most of our policies and guidelines but there is nothing major that I think needs to be changed with incredible urgency. I'll name three that I'm concerned about. This will no doubt annoy people who favor all three policies but such is life.
First, I think the fair-use guidelines could possibly use some tweaking to allow slightly more fair-use, but not much can be done with that without a change at the foundation level (and making any concrete proposals in this regard would require me to think about the issue a lot more than I have).
Second and more concretely, I'd like to see the BLP courtesy deletion changed so that it explicitly could not apply to willing public figures. People who have become involved in politics or other areas to the point where they are a public figure do not have a right to request courtesy deletion since they have willing opened themselves up to public scrutiny. Furthermore, not having articles on such people is obviously depriving people of coverage.
Third, I'd like to consider our inclusion policies (especially WP:N) to allow some degree of inherited notability, especially for characters in books or other works that are notable. Not so much as to make us automatically have an article on each one, but enough such as that we might not WP:FICT as strictly.

2. Can you point to a dispute (could have been at ArbCom or Mediation, or even on a talk page) that you've gone into (as an involved party or 3rd party) with a strong opinion, but had that opinion change in the course of discussion?

This is difficult since I don't hold strong opinions on many things on Wikipedia (indeed, if you look at many AfDs and DRVs you will see me repeatedly strike-out my previous comments as I change my position). One recent example where I had what might have been a strong opinion that changed was in the Martinphi-SA arbitration case. As you can see at the work shop page I strongly thought that SA's sockpuppetry while unfortunate was not abusive. I changed my mind when the evidence showed him using multiple accounts at once to change the tenor of a discussion.

Electing arbitrators: 1. Why did you enter this process over a week after it started? (I know it sounds accusatory, really isn't meant that way) Is the extended nomination period somehow unfair? Without specific reference to yourself or other candidates, what qualities, characteristics, or experiences do you think we should be looking for in an arbitrator?

I entered the nomination process a weak after it started because I wasn't sure I was going to run. I wasn't considering a run seriously until about a week before the nomination period opened when I then started receiving multiple emails saying that I should consider running. I then bounced the idea off of various users I trust and consulted various people in meatspace about whether they thought I had time for it. I then spent some time crafting my nomination statement which also took up 2 or 3 days.
As to the second part of the question- I don't see what could be unfair about the nomination period. If you could expand on the concern I might be able to answer it in more detail.
As to the third part, I think that important qualities in an arbitrator are patience, a thick skin, and experience with dispute resolution and various sorts of Wikipedia experience. One important form of experience is OTRS and related work. A number of candidates such as Deskana and Phil Sandifer have extensive experience with that which I see as a very good quality in an admin. I have experience informally helping when OTRS related issues have come on articles I already have on my watchlist or in a few other circumstances.

2. Last year the community nominated what looked like a solid bunch of Arbitrators. Yet 10 months later it turns out that several had very spotty levels of ArbCom activity. Do you think that this was at all predictable? And if so, how?

I'm not sure how to answer this. It might be possible to look at the less active arbitrators and pick out features they have in common, but there aren't any that stand out to me. Working as an arbitrator is clearly exhausting work that gets little thanks, and I suspect that in some cases the arbitrators were not as ready for it as they thought they would be. I don't know any way to foresee which arbitrators this will be a problem with.

You: 1. I've seen it written that to be a good arbitrator, a WPian needs to first be a good editor. Do you agree or disagree with the sentiment? How do you consider your own editing?

I'm not sure I agree or disagree with the sentiment. What constitutes a good editor? Most definitions of being a "good editor"(and I suspect that implied in the question) focus on work in mainspace. One definition of a good editor is an editor who has brought many articles up to good or featured status. Since ArbCom rarely makes content decisions [1] that shouldn't matter much. On the other hand, if someone has not made many good edits to a large variety of article they will likely not have the breath of experience necessary to understand some of the issues in front of the ArbCom. Another definition of a good editor is someone who has edited many controversial articles and helped people come to consensus about them. That might be more helpful for what an arbitrator does. There are other possible definitions of a "good editor" that focus less on mainspace but if we include them the number of possible definitions rapidly expands. Feel free to add a follow-up question with a more specific definition of good editor and I'll try to answer it.

2. Are you 15? 25? 55? Are you a student? Do you have an occupation that lends itself to allowing you time to be involved in ArbCom?

I am a student. I'm close to 25. I don't however think that an editor's age has any bearing on whether the editor would make a good arbitrator. Look at what the editor has done on Wikipedia. Look at the editor's statement and the editor's answers to questions. Judging by age makes in meatspace where we don't have the ability to examine all of someone's qualities in detail so we need to use somewhat reliable proxies. Wikipedia is transparent so we don't have much of that issue. As to the second part of the question, yes my occupation(student) does lend me to have time to be involved in ArbCom.

Thank you for indulging so many questions.

Question from Ultraexactzz

Best wishes in your candidacy, and in your tenure on the committee should you be elected. I'm asking this question to most of the candidates, so I apologize in advance if you've already answered a similar question from another editor.

Some background. I was an avid reader of the encyclopedia until December 2005, when I decided to begin editing. I had started to delve into the workings of the project, reading about AfD's and the ANI and, most interestingly, the work of the Arbitration Committee. When elections came around in December 2005/January 2006, I thought that a fresh perspective might be of value to the committee. So, in my haste to pitch in, I made my 13th edit (!) by nominating myself to the Arbitration Committee.

Needless to say, it did not go well.

However, I did find some editors who supported my candidacy on moral grounds, offering encouragement and concuring that a different perspective was of value in the committee's work. Looking back, it got me thinking, as this round of elections begins: What is the most valuable trait for an arbitrator? Your statement and answers to other questions will address this at length, I'm sure, but if you had to distill the essence of being an effective arbitrator into one word, what would that word be? ZZ Claims ~ Evidence 13:18, 13 November 2007 (UTC) reply

Josh doesn't think there is a single word to distill it into aside from illeism. Joking aside, I don't think there's a good single word, but if I had to make a small list I'd say 1) patience(which I have a lot of), 2)attention span (which I'll be honest I don't always have the best of), 3) experience with Wikipedia (which I have a lot of).

Question from Wikidudeman

In my experience, many larger arbitrations seem to suffer from the fact that the arbitrators do not spend as much time on examining the evidence and statements as they should be spending. Examples of problems that arise would be proposals not being used or relevant issues not even being addressed. This is probably due to the large backlog and caseload. What would you do to ensure that all arbitrations are ended efficiently and fairly and that all issues and concerns are addressed and all needed remedies met? Wikidudeman (talk) 14:48, 13 November 2007 (UTC) reply

First, let me disagree with one premise of the question: "proposals not being used" is not necessarily evidence that the ArbCom wasn't paying enough attention. For any given ArbCom at least the half the proposals in the workshop are simply bad. And for each good proposal there are often multiple variations of it proposed. Now, issues not being addressed is a more serious problem which does in many examples due to the arbitrators not examining the evidence sufficiently. As to this, I've already addressed in my statement and in prior questions some steps I'd take. To expand on that, I think one good idea is for individual cases for a few Arbitrators (maybe 2 ) to agree to focus in particular on that case and then to give the other arbitrators some summaries and pointers of what to look at. This would work more efficiently than having each arbitrator look at each detail of every single case. Of course, one will need to be careful to make sure that the few looking in detail don't miss anything, but it should help matters. (Disclaimer, I actually don't know much about how the ArbCom is currently dividing work informally but the impression I get is that they aren't doing much in that regard).

Question from Ragesoss

In the Wikipedia context, what is the difference (if any) between NPOV and SPOV (scientific point of view)?-- ragesoss 17:19, 13 November 2007 (UTC) reply

This is a complicated question that I've given a lot of thought to. Unfortunately, the thoughts aren't very organized, so I'll see what I can do. A quick preamble: WP:SPOV is a long rejected proposal to write from SPOV rather than NPOV. However, when people discuss the differences between them they aren't generally paying much attention to what that proposal said but rather to the more general notion of SPOV. Now moving on:
Most us agree that there is a difference between the two. I don't think that the difference is in general clear. For some topics, like say Reaction (physics) or electron, SPOV and NPOV versions of the article will be identical. However, for some topics, SPOV is meaningless. For example, it doesn't make sense to speak of an SPOV version of American Idol. And then there are articles where the SPOV version will differ from a NPOV, for example Homeopathy. An SPOV version of that article would like say something along the lines of "Homeopathy is a long debunked idea. It might be interesting for sociologists and psychologists to investigate why the topic keeps being brought up" followed by a few sources. Obviously, an article that said that would not be NPOV. On the other hand, there are issues of undue weight, and so for example Evolution which has an overwhelming scientific consensus gives only a small mention to views which disagree with it.
So where is the line for when SPOV does in fact equal NPOV? Suppose we had a reliable source that mentioned a person who thought that all the Wolbachia in the world was linked using tachyons to communicate as part of a hivemind working with the Illuminati and sentient Jewish gyroscopes to use alien device made by Xenu found in the Bermuda triangle to go back in time and frame Osama bin Laden for 9/11. [2] Would we need to mention it in the article Wolbachia? I think not. Well then, what if the one reliable reference was the New York Times or the Wall Street Journal? What if the New York Times did an article about this person? What if the person who had the idea was otherwise notable? Clearly for every major biology topic we don't need mention when we mention evolution "oh and by the way there are people who don't believe this" but we'd likely be remiss if we didn't have some mention at Evolution that there are people who don't accept it. These aren't easy issues and I'm not sure where to draw the line. There may not be a clear line.
However, the interaction is even more complicated because it isn't always clear if there is a single SPOV or an SPOV at all. As Durova noted in her well-thought out answer to the same question, it isn't always clear that there is an SPOV. As Durova noted, doctors in different countries can have radically different but still scientific attitudes about various symptoms and diseases. [3]
What Durova's brings up is not the only problem of this sort. Consider Electronic voice phenomena(EVP), where no serious scientists have bothered investigating the matter because from the general meta-patterns(I'd say meta-paradigm but that's really pretentious) of how modern science sees how the world works, the idea is improbable and a few moments of critical thinking only make it seem even less likely. So there are not many reliable sources to present the "SPOV" and yet we have a clear idea of what the SPOV is: Something close to "This is a waste of time. If these entities can change static to make hard to understand signals why can't they simply manipulate individual bits? There is no satisfactory explanation for why this supposedly works for both digital and analog static. For further information please see Pareidolia." So when people mean SPOV they don't really mean SPOV in the sense of what science has actually said about the matter, but rather what they expect science would say if scientists investigated the topic after they apply methodological naturalism and certain types of critical thinking. At some level this is almost unfortunate; the article on EVP needs to be almost completely credulous because scientists haven't taken the subject seriously at all and so has only two short paragraphs devoted to the SPOV. So precisely because the SPOV is so negative ( to the point of dismissal) the article has almost no sources to present about the SPOV. If one looks at EVP and Intelligent design one notes an interesting pattern which holds for many fringe science and pseudoscience topics; a topic doesn't get a response from the scientific community unless it has made enough inroads somewhere (either with scientists or lay people or claimed experts) that scientists consider it either responding to, investigating or debunking. This means that our policies at some level force us to present less popular fringe subjects more credulously than more popular fringe subjects.
There's another issue; even for those people today who don't favor science much, there's a feeling that science is a dominant viewpoint that needs to have more attention paid to it than others. Thus, when Biblical literalists contrast their viewpoint they contrast it to the scientific consensus, not to the extremist Hindus espousing a literal interpretation of the Vedas or to that of the followers of Immanuel Velikovsky. So one might almost say that NPOV favors a large amount of weight given to the scientific viewpoint because all alternative viewpoints(for lack of a better term) consider some sort of SPOV to be their major competitor. I'm not sure I agree with that argument, but it is worth considering.
However, scientific viewpoints do and should get a lot of weight for one simple reason: science is very good at producing reliable sources. Since the primary method of deciding how much weight to give something is how many reliable sources we have on a topic, this naturally gives science a large amount of weight. But even this raises questions: for example, Answers in Genesis and the Institute for Creation Research have both constructed what they claim are scientific journals, such as the "Journal of Creation" which has pro- Young Earth creationism articles which they claim are peer reviewed. Critics would likely label this work Cargo cult science. Now, are we as Wikipedia to treat these articles the same way we would treat mainstream scientific work in journals? Right now the consensus is not to, but for every fringe or pseudoscience group that makes a specialized journal the question of how to treat their journals come up. And there cases less clear cut than those for the Young Earth Creationists such as those that support Cold fusion.
So where does this all put us? There are four points that we should take away: 1) SPOV is not NPOV 2) What is SPOV is complicated. 3) How SPOV interacts with NPOV is complicated. 4) In general, when there is a coherent SPOV it should and will generally be given much weight but precisely how much weight is unclear.

Points of View: When does including "notable" points of view become problematic for NPOV?

When I first came on to Wikipedia a year and a half ago the project was more centered around "Just the facts" - articles were more crafted around the who, the what, the when and the where, with some emphasis on the why. Of late, the why has taken on a dominant role in articles on contentious issues, with each side in the political spectrum putting forth their own "notable" mouthpiece to spin what the who, the what, the when and the where means. Do you think this is a positive development? Do you think this is educational, or do you think it makes Wikipedia another platform for the dichotomized public debate--that there are two sides to every issues, and two views--that is prevalent in American society?--David Shankbone 18:27, 17 November 2007 (UTC) reply

I'm going to disagree somewhat with the premise. For many contentious articles even the facts are in dispute. See for example some of the disputes that occurred on Beit Hanoun November 2006 incident. To some extent this is a sign of Wikipedia's maturity and is a positive development. Many topics now have the articles flushed out better and we've reach consensuses about how to present matters where there is factual disagreement. Some policies like the strengthened BLP make it more clear what to do when there are questionable facts. To some extent this phenomenon is a sign that Wikipedia is more in the public eye, and so partisan groups understand that if they can control Wikipedia's presentation of the why they can influence the hearts and minds of many. I think Wikipedia needs to be careful not to aggravate what you call the dichotomized public debate. However, Wikipedia must reflect what reliable sources say. If all the reliable sources say simply "Some experts say X. Some experts say ~X and say that anyone who says X is stupid or lying" there isn't much we can do about it. Wikipedia can only reflect what the reliable sources say. At the same time, NPOV should help people understand all sides, when there are multiple sides or understand continua when there are continua of opinions.

Question from I

  1. What, if anything, do you believe is wrong with the current arbitration process, and/or the committe? This includes anything related to the committee and its actions. If appointed, what do you intend to do to resolve these issues? I (talk) 17:51, 18 November 2007 (UTC) reply
I think I've answered this well-enough in my statement as well as my replies to Heimstern and Wikidudeman above. JoshuaZ ( talk) 03:03, 24 November 2007 (UTC) reply

Questions from Piotrus

1. Do you think an arbitrator should be active in all cases he has no conflict or interests in?

No. Activity in all cases one can be involved in is a likely recipe for burnout.
If the arbitrator is active, should he be expected to comment in workshop / arbcom discussion pages?

Define "expected" it could likely help but there's no point in commenting if one doesn't have anything useful to add that hasn't already been said. Transparence is good but me too comments are not that useful.

  1. Do you think some editors should be more equal than others? I.e. should incivility of experienced editor - one who registered years ago and wrote or contributed to many articles - be treated differently from incivility of a relative newcomer?
I've answered this question to a limited extent with my answer to jd2718's question 1. Although he focused on administrators the basic thrust of the answer applies to all experienced editors with one additional point: if an editor has been around for a long time and has been consistently violating WP:CIVIL or any other policy or guideline the likelyhood of rehabilitation is low.
  1. How can WP:CIV and similar issues be enforced? Should they be enforced as efficient as 3RR?
CIV can be enforced by blocks, paroles and removals of commenst as we do so already. Unlike 3RR, WP:CIV is not always clear cut nor cut. Therefore it can be enforced as efficiently or mechanically as 3RR (indeed, mechanical enforcement of 3RR isn't often a good thing either).

-- Piotr Konieczny aka Prokonsul Piotrus | talk 01:47, 23 November 2007 (UTC) reply

Question from AniMate

Arbitration is the last step in dispute resolution. However, first and foremost, we are here to work on an encyclopedia. Editing and adding to the project should be everyone's first priority. Can you point out some of your recent mainspace contributions that you are most proud of? AniMate 12:06, 23 November 2007 (UTC) reply

My answer to this question will likely be not that impressive since I tend to poke around mainspace, not concentrating on any single article for that long. One article I've been recently involved with was Francis J. Beckwith‎ where the article subject has recently shown up and objected to some of the language and I've been working to resolve those. I've also been working on xkcd replacing original research about the comic with cited references. As you can see by checking my contribution list, the editing I've done is varied. JoshuaZ ( talk) 01:22, 26 November 2007 (UTC) reply

Question from Wanderer57

Based on ‘Request for comment on user conduct’ processes that you have followed closely, how would you rate them in terms of fairness to the accused?

(Just to be clear. Some candidates wondered if my question was "aimed at them". I'm asking all candidates the same generic question; it is not aimed at anyone.)

Thanks, Wanderer57 ( talk) 15:14, 24 November 2007 (UTC) reply

First, the RfC process is an attempt to get people to listen to criticism and to reform, so referring to the subject as the "accused" is not helpful. That out of the way: I'm divided about the RfC process. Sometimes is degenerates into what looks like mobs, and RfCs will sometimes draw out everyone who has a grievance with a given editor. However, when the community does support the subject's actions such as Wikipedia:Requests_for_comment/Bishonen it is made clear. A more serious issue is that the process rarely does anything. It is useful for taking some load off of arb-com, but it isn't much more than that. And I'm not convinced it really reduced the Arb-Com's load all that much. JoshuaZ ( talk) 01:35, 26 November 2007 (UTC) reply

Question from Cla68

So that it won't look like I'm targeting anyone in particular, I'm asking this question of all the candidates. Were you a recipient on the email list used by Durova to distribute her evidence used to wrongfully block !! as detailed in this ArbCom case? Cla68 ( talk) 00:50, 27 November 2007 (UTC) reply

I'm highly tempted not to answer this question as it is completely and utterly irrelevant and fosters unnecessary drama. Off-wiki communications are a standard part of Wikipedia discussions and are especially necessary when dealing with determined troublemakers like those who frequent WR and ED. That said, no I was not. I have not seen any serious wrong done here. Durova made an incorrect block, after some discussion the block was overturned. Almost every single admin has made at least one bad block (I've made at least 4). This matter is being blown out of proportion and is a complete waste of time.

Questions from Rschen7754

1. What are your views regarding debates such as WP:RFAR/HWY and WP:SRNC? (In terms of dispute resolution).

The ongoing highways disupute was one of the most amazingly lame disputes I've ever seen and wasted a tremendous amount of time. I didn't pay much attention to the dispute preciesely for that reason. However, my general impression is that the final result (the poll and related issues) handled at best could be handled a highly contentious dispute.

2. a) What is the purpose of a WikiProject? Do you believe that WikiProjects b) own articles or c) can enforce standards (such as article layout) on articles?

Wikiprojects exist to coordinate editing. I'm generally worried when wikiprojects attempt to own articles or ayyrmpy to enforce their own article layout standard. Note that this is distinct from a general consensus of editors involved with a type of article reaching a consensus about how layout and other issues should be standardized. That is in general fine, and if most of the editors involved in such a consensus are part of a wikiproject that's ok. However, Wikiprojects do not in general have any special power.

3. Do you believe that parent WikiProjects have the right to impose standards (such as article layout) on child WikiProjects? (Case in point: WP:USRD and its state highway projects)

See question 2 above. Since Wikiprojects have no special power to impose standards on article space it doesn't make sense to discuss them imposing article layout on their child projects.

4. a) What is your definition of canvassing? b) Does it include project newsletters or IRC?

Canvassing is a complicated issue and I'm not sure that I can give a short answer. I would suggest a definition along the lines of "Deliberately asking for people of one opinion to chime in in a discussion or vote without alerting people with opposing opinions."

However, this definition can be too broad on occasions. If for example a single editor said to another editor "let me know when you are running for admin" it wouldn't be canvassing to alert the editor in question. And if that happened with 2 or 3 editors it wouldn't be either. Where exactly one draws lines for this sort of thing isn't always clear.

5. a) In terms of vandalism and good faith but horrible edits, where do you draw the line? (scenario: an editor makes a mess of articles that cannot easily be fixed). b) Should blocks, protects, and / or rollbacks be in order?

As to a, it really depends on the circumstances. If it doesn't appear to be vandalism then communicating with the editor is probably a good idea. Blocks should not be used unless multiple attempts to communicate fail. Rollback might also be used if cleaning up would be very difficult otherwise. However, these are my personal opinions (and are vague at that); I don't claim that there's any real policy basis behind this. If someone else drew the line elsewhere I wouldn't have a problem with that.

Thank you. -- Rschen7754 ( T C) 06:58, 27 November 2007 (UTC) reply

Question from Blue Tie

1. Can/should Arbcom create wikipedia policy? Or develop a proposed policy for community vote?

Arbcom should not in general create policy. However, in practice the ArbCom does sometimes do so, for example deciding that certain types of conduct are unacceptable even when no policy said so. I'd prefer that this sort of policy making be kept to a minimum and would strongly not favor retroactive punishment for policy violation.

2. Do you intend to help create or propose wikipedia policy as an Arbcom member? -- Blue Tie 13:19, 30 November 2007 (UTC) reply

I have constructed policy issues in the past as a general Wikipedian and intend to do so in that capacity whether or not I am elected to the ArbCom. As I said before, ArbCom should not take part in writing policy unless absolutely necessary.

Question from SilkTork

How would you vote on this proposed principle: "While anyone may edit Wikipedia without the need to register, that meta-editing activities such as voting in an ArbCom Election are best protected by registering than by sleuthing". SilkTork * SilkyTalk 17:44, 1 December 2007 (UTC) reply

I disagree with this proposal in part and agree with it in part. First, there is no easy mechanism to create a registration process that would not be circumventable. Having the Foundation do so would be costly, unwieldy and get the foundation involved to an extent that could make it up unacceptably vulnerable from a legal perspective. Almost any registration system (such as having voters' real names on file somewhere) would also require people who registered to give up some of their privacy. Furthermore, our most common forms of voting is RfA and that had way too many people involved in it. Similarly, AfDs have way too many people for this to be practical. I also object to the labeling of "sleuthing" as "security by obscurity". We generally use so called "sleuthing" to pick up socks of known repeated disrupters. Many people have little tell-tails about how they write and edit. Letting such details in public is problematic since the people can then adjust. But in general, such methods when used properly and when backed up by checkuser and other evidence can be very useful. Now, given the much smaller number of people who vote in the ArbCom election, doing something similar restricted solely to people who want to vote might be more workable. But really insidious socks aren't likely to change something like the ArbCom results, but are more likely to do serious damage at AfD or other areas. JoshuaZ ( talk) 21:13, 13 December 2007 (UTC) reply

Questions from Irpen

The questions below refer to the issues of ArbCom's integrity and transparency that needs to be maintained despite the universally accepted view that certain things should remain private.

Mailing list

Arbitrator's private mailing list, known as Arbcom-l and the arbitrators only IRC channel may obviously include information that cannot be made public under any circumstances. Additionally, being aware of the intra-ArbCom communication may give case parties an obvious advantage over their opponents. Who do you think should have access to such a list besides current arbitrators whose community trust has been confirmed in election that took place within the last 3 years? Should it include users that where never voted on? Should it include users who were voted 4, 5 or more years ago? Should users who are parties of the case, comment on the case, present evidence on the case, be allowed to have read access to the list where the case is discussed by the decision makers?

Secret evidence and secret communication of arbitrators with non-arbitrators

What is your opinion about the parties of the case (or anyone) contacting arbitrators privately about the case? This is not an hypothetical issue and it has been brought up in past cases. The obvious drawback is that if charges are brought secretly, the accused cannot see them and respond. Would you support an amendment of the arbitration policy that would prohibit parties from writing to arbitrators privately in relation to the cases? Giving evidence that has to be private due to its sensitive nature would of course be exempted but should this be the only exception?

Recusals

Arbitrators who are parties of the case or have an involvement with the case parties that can reasonably be considered to affect their impartiality are expected to recuse. What involvement constitutes the ground for a recusal has traditionally been left to the arbitrators' own discretion, except for obvious cases when arbitrators themselves are case parties. While recused arbitrators, especially the case parties, are allowed to take an active part in cases, collect, present and discuss evidence at the case pages, the same way as ordinary parties, they retain the opportunity to read the thoughts of other arbitrators at Arbcom-L and respond to those privately. It is technically difficult to exclude arbitrators from communication on a case they are involved. But would you support a prohibition for such arbitrators to discuss the case with other arbitrators through the private communication channels, except when submitting evidence whose nature warrants non-publicity?

Community oversight over the arbitration policy

Policies are written by the community and not by the ArbCom. However, at some point the ArbCom made it clear that the arbitration policy is exceptional in this respect and that the ArbCom intends to control the main policy that governs its own action rather than be governed by the policy written by the community. Would you support returning the control of the ArbCom policy back to the community or should the ArbCom write its policy itself?

Question from Mrs.EasterBunny

As a member of ArbCom, would you place more emphasis on content or behavior? For example, in the http://en.wikipedia.org/wiki/Wikipedia:Requests_for_arbitration/SevenOfDiamonds case, there is voluminous discussion on whether SevenofDiamonds is really NuclearUmpf, but no discussion on what got NuclearUmpf banned in the first place. If SevenofDiamonds=NuclearUmpf, then this is a behavioral problem but doesn't have to be a content problem. If SevenofDiamonds edit was reasonable (I have not researched it) would it make a difference?

The above may not be the best example but it's one that I recently saw because I can't remember the parties involved in similar cases. On occasion, I have seen an editing admin block someone because of a dispute in editing an article that both of them are editing and the block seemed questionable because there is no overt POV. The blocked editor then probably feels the block is unjustified and creates a sock. Many times, people running for WP office will cite a clear cut case of someone with bad editing and bad behavior. However, what if there is good editing and improper block (which would point to admin misconduct about content), followed by sock creation justified because the block was improper (which would point to editor misconduct about behavior)? Does the first crime excuse the second? Or is the second one crime much more serious and punishable? (This is not an easy question because excusing the first crime by the admin would tend to increase the workload of ArbCom because it allows admin to do a lot with less oversight. However, excusing the second crime might seem to encourage socks). Mrs.EasterBunny ( talk) 20:18, 5 December 2007 (UTC) reply

Question from wbfergus

What is your position on the following?

  • A policy page has had a very active discussion for many months. All sides (loosely termed 'pro-change', 'anti-change' and 'issue-specific') of proposed changes have made their cases back and forth numerous times. The 'pro-change' group is mainly users, with a few Admins. The 'anti-change' group is mainly Admins (including those who helped write the policy over the years) and a few users. The 'issue-specific' group is a mixed collection of users and Admins, but mainly users. All three groups constitute around 40-50 people total, per announcements on the Village Pump and related policies, to garner more widespread community involvement either way.
  1. After numerous discussions, and comments over a span of several days to several weeks on specific issues, what should constitute a consensus? 60%, 75%, 90%, or unanimous approval?
  2. If around 75% agree to a change, is it appropriate for Admins (especially those who helped write the policy) to revert changes and protect the page from further edits against their approval?
  3. Is it appropriate for 6 or 7 Admins to more or less block changes to a policy through protection and reverts, when very active discussions have been ongoing and the majority of those participating constructively (not just saying "No" or "Oppose" without constructive comments) agree to changes?
  4. Would it be appropriate for such a policy page which does clearly have a disputed section to have a tag in that section stating that section is under dispute and to participate on the talk page?
  5. Should policies solely dictate acceptable and unacceptable content, behaviour, etc., or should they also define Wikipedia-specific terms and definitions (without stating so) that conflict with usage in different disciplines, or should such terms and definitions be more appropriately suited in a guideline linked to and from the policy?
  6. Do you agree that policies are meant for enforcement or 'enforceable actions', while guidelines are meant to give guidance?

wbfergus Talk 15:35, 12 December 2007 (UTC) reply

For the record, I feel that I need to close my questions to all candidates, as one of the editors in the above 'subject' has filed an ArbCom request. As such, it could be interpreted as unseemly or whatever for these issues to be addressed in this forum. I was in the process of cancelling my questions and replying in an RfC and the related ArbCom request when I had to leave to take my wife to a Dr. appointment, so pardon the delay in cancelling this. wbfergus Talk 21:15, 13 December 2007 (UTC) reply

References

  1. ^ I'm not going to pretend that the ArbCom never makes content decisions even though that is the standard line. The BLP/Badlydrawnjeff decision for example led to what amounted to essentially content decisions for example. Furthermore, when the ArbCom bans or restricts editors who have a certain POV on an article it is going to have content results albeit more indirectly.
  2. ^ I constructed this example from a variety of different conspiracy theories, each one of which I've seen mentioned in at least one reliable source with the exception of the gyroscope element which I borrowed from xkcd. My intent here isn't just humor. We need to be able to decide how much weight to give to subjects and people do inevitably include the giggle factor. If they can't take a subject seriously they will be less likely to give it as much weight in an article. This can lead to problems. I've seen AfDs where the sole cause of the AfD was the nominator considering the subject to be too ridiculous to merit an article. For the relevant xkcd see "Gyroscopes(#332)". xkcd. Retrieved 2007-11-23..
  3. ^ See Wikipedia:Arbitration_Committee_Elections_December_2007/Candidate_statements/Durova/Questions_for_the_candidate#23_17 for Durova's excellent and concise description of this problem.

Question from Pinkville

Wikipedia is a community that produces and maintains a (still-nascent) encyclopaedia. This community has particular social and political structures that define it and that, presumably, affect the character, quality, and depth of its encyclopaedic output. Can you briefly summarise some political and social aspects of the Wikipedia community that you consider important or noteworthy, that perhaps need to be challenged or developed? How does the structure of Wikipedia encourage or inhibit access to decision-making and issues of power/control? Or does any of that matter? And what are the implications for the Arbitration Committee and its members? Pinkville ( talk) 22:10, 12 December 2007 (UTC) reply


Videos

Youtube | Vimeo | Bing

Websites

Google | Yahoo | Bing

Encyclopedia

Google | Yahoo | Bing

Facebook